2018 AMC 12A Problem 21: Which of the given polynomials has the greatest real root?

The name of the pictureThe name of the pictureThe name of the pictureClash Royale CLAN TAG#URR8PPP











up vote
0
down vote

favorite
1












Which of the following polynomials has the greatest real root?
$textbf(A) x^19+2018x^11+1 qquad textbf(B) x^17+2018x^11+1 qquad textbf(C) x^19+2018x^13+1 qquad textbf(D) x^17+2018x^13+1 qquad textbf(E) 2019x+2018$



The only root that I was able to find was the last one. I have no idea what to do from here. Does anyone have some ideas?







share|cite|improve this question






















  • The problem, including three solutions, can also be found here: artofproblemsolving.com/wiki/…
    – Jendrik Stelzner
    Aug 28 at 0:23














up vote
0
down vote

favorite
1












Which of the following polynomials has the greatest real root?
$textbf(A) x^19+2018x^11+1 qquad textbf(B) x^17+2018x^11+1 qquad textbf(C) x^19+2018x^13+1 qquad textbf(D) x^17+2018x^13+1 qquad textbf(E) 2019x+2018$



The only root that I was able to find was the last one. I have no idea what to do from here. Does anyone have some ideas?







share|cite|improve this question






















  • The problem, including three solutions, can also be found here: artofproblemsolving.com/wiki/…
    – Jendrik Stelzner
    Aug 28 at 0:23












up vote
0
down vote

favorite
1









up vote
0
down vote

favorite
1






1





Which of the following polynomials has the greatest real root?
$textbf(A) x^19+2018x^11+1 qquad textbf(B) x^17+2018x^11+1 qquad textbf(C) x^19+2018x^13+1 qquad textbf(D) x^17+2018x^13+1 qquad textbf(E) 2019x+2018$



The only root that I was able to find was the last one. I have no idea what to do from here. Does anyone have some ideas?







share|cite|improve this question














Which of the following polynomials has the greatest real root?
$textbf(A) x^19+2018x^11+1 qquad textbf(B) x^17+2018x^11+1 qquad textbf(C) x^19+2018x^13+1 qquad textbf(D) x^17+2018x^13+1 qquad textbf(E) 2019x+2018$



The only root that I was able to find was the last one. I have no idea what to do from here. Does anyone have some ideas?









share|cite|improve this question













share|cite|improve this question




share|cite|improve this question








edited Aug 28 at 0:21









Jendrik Stelzner

7,63121037




7,63121037










asked Jun 10 at 1:20









Dude156

17312




17312











  • The problem, including three solutions, can also be found here: artofproblemsolving.com/wiki/…
    – Jendrik Stelzner
    Aug 28 at 0:23
















  • The problem, including three solutions, can also be found here: artofproblemsolving.com/wiki/…
    – Jendrik Stelzner
    Aug 28 at 0:23















The problem, including three solutions, can also be found here: artofproblemsolving.com/wiki/…
– Jendrik Stelzner
Aug 28 at 0:23




The problem, including three solutions, can also be found here: artofproblemsolving.com/wiki/…
– Jendrik Stelzner
Aug 28 at 0:23










2 Answers
2






active

oldest

votes

















up vote
2
down vote



accepted










We start by observing the terms in options A to D. The leading term is degree 19 or 17 while the second term is degree 11 or 13. The numbers caught my eyes because $19-17=2$ and $13-11=2$ reminds me of factoring quadratics. I decide to compute
$$A-B=x^19-x^17=x^17(x^2-1)$$
$$A-C=2018x^11-2018x^13=2018x^11(1-x^2)$$
$$C-D=x^19-x^17=x^17(x^2-1)$$



Steps for the answer:




None of the options have positive real roots. The largest root then has to lie in $(-infty,0]$. Now, when $xin(-1,0)$, $A-B>0$, $A-C<0$, $C-D>0$, so $B>A>C>D$. Because $A$ to $D$ are all monotonously increasing on $xin(-1,0)$ (check the derivatives), we see that $B$ has the largest real root in the interval $(-1,0)$. When $x=-frac20182019$, $B$ is negative while $E$ is $0$, so the root of $B$ has to be on the right of the root of $E$.




Gist:



enter image description here



In the figure, the green function is always larger than the red function. If in an interval, both functions are monotonously increasing and the green one is always larger, then the root of the green function is smaller. If in an interval, both functions are monotonously decreasing and the green one is always larger, then the root of the green function is larger.






share|cite|improve this answer


















  • 1




    — I don't get this part — "A−B>0 , A−C<0, C−D>0, so B>A>C>D". Shouldn't it be B<A<C, and C>D, and I'm also not sure how we can compare all four simultaneously... Could you explain please?
    – jjhh
    Aug 27 at 17:17










  • @jjhh Ahh... I made a mistake there. I'll salvage my answer soon.
    – Frenzy Li
    Aug 27 at 22:00

















up vote
2
down vote













Let us first use computer power to have the verdict, code:



polynomials = ( x^19 + 2018*x^11 + 1, 
x^17 + 2018*x^11 + 1,
x^19 + 2018*x^13 + 1,
x^17 + 2018*x^13 + 1,
2019*x + 2018 )
for p in polynomials:
print ( "%s has only real root at %s"
% ( p, p.roots(ring=AA, multiplicities=False)[0] ) )


Results:



x^19 + 2018*x^11 + 1 has only real root at -0.5006711240348649?
x^17 + 2018*x^11 + 1 has only real root at -0.500670857826262?
x^19 + 2018*x^13 + 1 has only real root at -0.556896901348879?
x^17 + 2018*x^13 + 1 has only real root at -0.5568954928592386?
2019*x + 2018 has only real root at -0.9995047052996532?


(sage was used, free software for all mathematical purposes, it can even be "infected" with non-free software, if the user possesses the one or the other compatible choice.)



Note: The above numbers are printed with a question mark at their end, but sage can does in fact exact computations with such numbers.



And "the winner is..."



x^17 + 2018*x^11 + 1


Let us prove this by human means.




  • First of all, let us show that all roots are in the interval $[-1,0]$. Indeed, all five polynomials have



    • a negative value in $-1$, it is $-2018$ for the first four, and $-1$ for the last one,

    • a positive value in $0$, it is $+1$ for the first four, and bigger for the last one.

    So each polynomial has a root in the interval $[-1,0]$. The first derivative for each given polynomial is $>0$ on $Bbb R$, so the given polynomial functions are strictly increasing, from $-infty$ (when the argument tends to this limit) to $+infty$ (when the argument tends to this limit). So each given polynomial has the root in the interval $[-1,0]$.



  • The last polynomial has the root $-2018/2019<-9/10=-0.9$, and if we plug in this value, $0.9$, in any $P$ under the other four we get a negative number, since $0.9^13approx 0.25dots$, and $0.9^11approx 0.31dots$, so the term with the coefficient $2018$ "wins", $P(-0,9)<2018cdot frac 14+1<0$.


Let now $ain(-1,0)$ be the root of
$$ P=x^17 + 2018x^11 + 1 ,$$
and let
$$ Q=x^A + 2018x^B + 1$$
be an other polynomial in the list, so $Ain17,19$,
$Bin11,13$ and either $A>17$, or $B>11$.
We have
$$
-1<a^11<a^13<a^17<a^19<0
,
$$
so
$$Q(a) > P(a)=0 ,$$
so the root of $Q$ is in the interval $[-1,a]$, being thus smaller than $a$.



$square$




Ths problem is solved, but it is always good to give a plot for the actors, in our case for the four polynomials, in the interval that hurts, here first $[-1,0]$.



First plot, four polynomials in $[-1, 0]$



...then $[-0.6, 0]$.



Second plot, four polynomials in $[-0.6, 0]$



The following code was used:



sage: p1, p2, p3, p4 = [ plot( p, (-1,0) ) for p in polynomials[:4] ]
sage: (p1+p2+p3+p4).show()
Launched png viewer for Graphics object consisting of 4 graphics primitives
sage: p1, p2, p3, p4 = [ plot( p, (-0.6,0) ) for p in polynomials[:4] ]
sage: (p1+p2+p3+p4).show()
Launched png viewer for Graphics object consisting of 4 graphics primitives





share|cite|improve this answer




















    Your Answer




    StackExchange.ifUsing("editor", function ()
    return StackExchange.using("mathjaxEditing", function ()
    StackExchange.MarkdownEditor.creationCallbacks.add(function (editor, postfix)
    StackExchange.mathjaxEditing.prepareWmdForMathJax(editor, postfix, [["$", "$"], ["\\(","\\)"]]);
    );
    );
    , "mathjax-editing");

    StackExchange.ready(function()
    var channelOptions =
    tags: "".split(" "),
    id: "69"
    ;
    initTagRenderer("".split(" "), "".split(" "), channelOptions);

    StackExchange.using("externalEditor", function()
    // Have to fire editor after snippets, if snippets enabled
    if (StackExchange.settings.snippets.snippetsEnabled)
    StackExchange.using("snippets", function()
    createEditor();
    );

    else
    createEditor();

    );

    function createEditor()
    StackExchange.prepareEditor(
    heartbeatType: 'answer',
    convertImagesToLinks: true,
    noModals: false,
    showLowRepImageUploadWarning: true,
    reputationToPostImages: 10,
    bindNavPrevention: true,
    postfix: "",
    noCode: true, onDemand: true,
    discardSelector: ".discard-answer"
    ,immediatelyShowMarkdownHelp:true
    );



    );













     

    draft saved


    draft discarded


















    StackExchange.ready(
    function ()
    StackExchange.openid.initPostLogin('.new-post-login', 'https%3a%2f%2fmath.stackexchange.com%2fquestions%2f2814099%2f2018-amc-12a-problem-21-which-of-the-given-polynomials-has-the-greatest-real-ro%23new-answer', 'question_page');

    );

    Post as a guest






























    2 Answers
    2






    active

    oldest

    votes








    2 Answers
    2






    active

    oldest

    votes









    active

    oldest

    votes






    active

    oldest

    votes








    up vote
    2
    down vote



    accepted










    We start by observing the terms in options A to D. The leading term is degree 19 or 17 while the second term is degree 11 or 13. The numbers caught my eyes because $19-17=2$ and $13-11=2$ reminds me of factoring quadratics. I decide to compute
    $$A-B=x^19-x^17=x^17(x^2-1)$$
    $$A-C=2018x^11-2018x^13=2018x^11(1-x^2)$$
    $$C-D=x^19-x^17=x^17(x^2-1)$$



    Steps for the answer:




    None of the options have positive real roots. The largest root then has to lie in $(-infty,0]$. Now, when $xin(-1,0)$, $A-B>0$, $A-C<0$, $C-D>0$, so $B>A>C>D$. Because $A$ to $D$ are all monotonously increasing on $xin(-1,0)$ (check the derivatives), we see that $B$ has the largest real root in the interval $(-1,0)$. When $x=-frac20182019$, $B$ is negative while $E$ is $0$, so the root of $B$ has to be on the right of the root of $E$.




    Gist:



    enter image description here



    In the figure, the green function is always larger than the red function. If in an interval, both functions are monotonously increasing and the green one is always larger, then the root of the green function is smaller. If in an interval, both functions are monotonously decreasing and the green one is always larger, then the root of the green function is larger.






    share|cite|improve this answer


















    • 1




      — I don't get this part — "A−B>0 , A−C<0, C−D>0, so B>A>C>D". Shouldn't it be B<A<C, and C>D, and I'm also not sure how we can compare all four simultaneously... Could you explain please?
      – jjhh
      Aug 27 at 17:17










    • @jjhh Ahh... I made a mistake there. I'll salvage my answer soon.
      – Frenzy Li
      Aug 27 at 22:00














    up vote
    2
    down vote



    accepted










    We start by observing the terms in options A to D. The leading term is degree 19 or 17 while the second term is degree 11 or 13. The numbers caught my eyes because $19-17=2$ and $13-11=2$ reminds me of factoring quadratics. I decide to compute
    $$A-B=x^19-x^17=x^17(x^2-1)$$
    $$A-C=2018x^11-2018x^13=2018x^11(1-x^2)$$
    $$C-D=x^19-x^17=x^17(x^2-1)$$



    Steps for the answer:




    None of the options have positive real roots. The largest root then has to lie in $(-infty,0]$. Now, when $xin(-1,0)$, $A-B>0$, $A-C<0$, $C-D>0$, so $B>A>C>D$. Because $A$ to $D$ are all monotonously increasing on $xin(-1,0)$ (check the derivatives), we see that $B$ has the largest real root in the interval $(-1,0)$. When $x=-frac20182019$, $B$ is negative while $E$ is $0$, so the root of $B$ has to be on the right of the root of $E$.




    Gist:



    enter image description here



    In the figure, the green function is always larger than the red function. If in an interval, both functions are monotonously increasing and the green one is always larger, then the root of the green function is smaller. If in an interval, both functions are monotonously decreasing and the green one is always larger, then the root of the green function is larger.






    share|cite|improve this answer


















    • 1




      — I don't get this part — "A−B>0 , A−C<0, C−D>0, so B>A>C>D". Shouldn't it be B<A<C, and C>D, and I'm also not sure how we can compare all four simultaneously... Could you explain please?
      – jjhh
      Aug 27 at 17:17










    • @jjhh Ahh... I made a mistake there. I'll salvage my answer soon.
      – Frenzy Li
      Aug 27 at 22:00












    up vote
    2
    down vote



    accepted







    up vote
    2
    down vote



    accepted






    We start by observing the terms in options A to D. The leading term is degree 19 or 17 while the second term is degree 11 or 13. The numbers caught my eyes because $19-17=2$ and $13-11=2$ reminds me of factoring quadratics. I decide to compute
    $$A-B=x^19-x^17=x^17(x^2-1)$$
    $$A-C=2018x^11-2018x^13=2018x^11(1-x^2)$$
    $$C-D=x^19-x^17=x^17(x^2-1)$$



    Steps for the answer:




    None of the options have positive real roots. The largest root then has to lie in $(-infty,0]$. Now, when $xin(-1,0)$, $A-B>0$, $A-C<0$, $C-D>0$, so $B>A>C>D$. Because $A$ to $D$ are all monotonously increasing on $xin(-1,0)$ (check the derivatives), we see that $B$ has the largest real root in the interval $(-1,0)$. When $x=-frac20182019$, $B$ is negative while $E$ is $0$, so the root of $B$ has to be on the right of the root of $E$.




    Gist:



    enter image description here



    In the figure, the green function is always larger than the red function. If in an interval, both functions are monotonously increasing and the green one is always larger, then the root of the green function is smaller. If in an interval, both functions are monotonously decreasing and the green one is always larger, then the root of the green function is larger.






    share|cite|improve this answer














    We start by observing the terms in options A to D. The leading term is degree 19 or 17 while the second term is degree 11 or 13. The numbers caught my eyes because $19-17=2$ and $13-11=2$ reminds me of factoring quadratics. I decide to compute
    $$A-B=x^19-x^17=x^17(x^2-1)$$
    $$A-C=2018x^11-2018x^13=2018x^11(1-x^2)$$
    $$C-D=x^19-x^17=x^17(x^2-1)$$



    Steps for the answer:




    None of the options have positive real roots. The largest root then has to lie in $(-infty,0]$. Now, when $xin(-1,0)$, $A-B>0$, $A-C<0$, $C-D>0$, so $B>A>C>D$. Because $A$ to $D$ are all monotonously increasing on $xin(-1,0)$ (check the derivatives), we see that $B$ has the largest real root in the interval $(-1,0)$. When $x=-frac20182019$, $B$ is negative while $E$ is $0$, so the root of $B$ has to be on the right of the root of $E$.




    Gist:



    enter image description here



    In the figure, the green function is always larger than the red function. If in an interval, both functions are monotonously increasing and the green one is always larger, then the root of the green function is smaller. If in an interval, both functions are monotonously decreasing and the green one is always larger, then the root of the green function is larger.







    share|cite|improve this answer














    share|cite|improve this answer



    share|cite|improve this answer








    edited Jun 10 at 2:34

























    answered Jun 10 at 2:24









    Frenzy Li

    2,81822345




    2,81822345







    • 1




      — I don't get this part — "A−B>0 , A−C<0, C−D>0, so B>A>C>D". Shouldn't it be B<A<C, and C>D, and I'm also not sure how we can compare all four simultaneously... Could you explain please?
      – jjhh
      Aug 27 at 17:17










    • @jjhh Ahh... I made a mistake there. I'll salvage my answer soon.
      – Frenzy Li
      Aug 27 at 22:00












    • 1




      — I don't get this part — "A−B>0 , A−C<0, C−D>0, so B>A>C>D". Shouldn't it be B<A<C, and C>D, and I'm also not sure how we can compare all four simultaneously... Could you explain please?
      – jjhh
      Aug 27 at 17:17










    • @jjhh Ahh... I made a mistake there. I'll salvage my answer soon.
      – Frenzy Li
      Aug 27 at 22:00







    1




    1




    — I don't get this part — "A−B>0 , A−C<0, C−D>0, so B>A>C>D". Shouldn't it be B<A<C, and C>D, and I'm also not sure how we can compare all four simultaneously... Could you explain please?
    – jjhh
    Aug 27 at 17:17




    — I don't get this part — "A−B>0 , A−C<0, C−D>0, so B>A>C>D". Shouldn't it be B<A<C, and C>D, and I'm also not sure how we can compare all four simultaneously... Could you explain please?
    – jjhh
    Aug 27 at 17:17












    @jjhh Ahh... I made a mistake there. I'll salvage my answer soon.
    – Frenzy Li
    Aug 27 at 22:00




    @jjhh Ahh... I made a mistake there. I'll salvage my answer soon.
    – Frenzy Li
    Aug 27 at 22:00










    up vote
    2
    down vote













    Let us first use computer power to have the verdict, code:



    polynomials = ( x^19 + 2018*x^11 + 1, 
    x^17 + 2018*x^11 + 1,
    x^19 + 2018*x^13 + 1,
    x^17 + 2018*x^13 + 1,
    2019*x + 2018 )
    for p in polynomials:
    print ( "%s has only real root at %s"
    % ( p, p.roots(ring=AA, multiplicities=False)[0] ) )


    Results:



    x^19 + 2018*x^11 + 1 has only real root at -0.5006711240348649?
    x^17 + 2018*x^11 + 1 has only real root at -0.500670857826262?
    x^19 + 2018*x^13 + 1 has only real root at -0.556896901348879?
    x^17 + 2018*x^13 + 1 has only real root at -0.5568954928592386?
    2019*x + 2018 has only real root at -0.9995047052996532?


    (sage was used, free software for all mathematical purposes, it can even be "infected" with non-free software, if the user possesses the one or the other compatible choice.)



    Note: The above numbers are printed with a question mark at their end, but sage can does in fact exact computations with such numbers.



    And "the winner is..."



    x^17 + 2018*x^11 + 1


    Let us prove this by human means.




    • First of all, let us show that all roots are in the interval $[-1,0]$. Indeed, all five polynomials have



      • a negative value in $-1$, it is $-2018$ for the first four, and $-1$ for the last one,

      • a positive value in $0$, it is $+1$ for the first four, and bigger for the last one.

      So each polynomial has a root in the interval $[-1,0]$. The first derivative for each given polynomial is $>0$ on $Bbb R$, so the given polynomial functions are strictly increasing, from $-infty$ (when the argument tends to this limit) to $+infty$ (when the argument tends to this limit). So each given polynomial has the root in the interval $[-1,0]$.



    • The last polynomial has the root $-2018/2019<-9/10=-0.9$, and if we plug in this value, $0.9$, in any $P$ under the other four we get a negative number, since $0.9^13approx 0.25dots$, and $0.9^11approx 0.31dots$, so the term with the coefficient $2018$ "wins", $P(-0,9)<2018cdot frac 14+1<0$.


    Let now $ain(-1,0)$ be the root of
    $$ P=x^17 + 2018x^11 + 1 ,$$
    and let
    $$ Q=x^A + 2018x^B + 1$$
    be an other polynomial in the list, so $Ain17,19$,
    $Bin11,13$ and either $A>17$, or $B>11$.
    We have
    $$
    -1<a^11<a^13<a^17<a^19<0
    ,
    $$
    so
    $$Q(a) > P(a)=0 ,$$
    so the root of $Q$ is in the interval $[-1,a]$, being thus smaller than $a$.



    $square$




    Ths problem is solved, but it is always good to give a plot for the actors, in our case for the four polynomials, in the interval that hurts, here first $[-1,0]$.



    First plot, four polynomials in $[-1, 0]$



    ...then $[-0.6, 0]$.



    Second plot, four polynomials in $[-0.6, 0]$



    The following code was used:



    sage: p1, p2, p3, p4 = [ plot( p, (-1,0) ) for p in polynomials[:4] ]
    sage: (p1+p2+p3+p4).show()
    Launched png viewer for Graphics object consisting of 4 graphics primitives
    sage: p1, p2, p3, p4 = [ plot( p, (-0.6,0) ) for p in polynomials[:4] ]
    sage: (p1+p2+p3+p4).show()
    Launched png viewer for Graphics object consisting of 4 graphics primitives





    share|cite|improve this answer
























      up vote
      2
      down vote













      Let us first use computer power to have the verdict, code:



      polynomials = ( x^19 + 2018*x^11 + 1, 
      x^17 + 2018*x^11 + 1,
      x^19 + 2018*x^13 + 1,
      x^17 + 2018*x^13 + 1,
      2019*x + 2018 )
      for p in polynomials:
      print ( "%s has only real root at %s"
      % ( p, p.roots(ring=AA, multiplicities=False)[0] ) )


      Results:



      x^19 + 2018*x^11 + 1 has only real root at -0.5006711240348649?
      x^17 + 2018*x^11 + 1 has only real root at -0.500670857826262?
      x^19 + 2018*x^13 + 1 has only real root at -0.556896901348879?
      x^17 + 2018*x^13 + 1 has only real root at -0.5568954928592386?
      2019*x + 2018 has only real root at -0.9995047052996532?


      (sage was used, free software for all mathematical purposes, it can even be "infected" with non-free software, if the user possesses the one or the other compatible choice.)



      Note: The above numbers are printed with a question mark at their end, but sage can does in fact exact computations with such numbers.



      And "the winner is..."



      x^17 + 2018*x^11 + 1


      Let us prove this by human means.




      • First of all, let us show that all roots are in the interval $[-1,0]$. Indeed, all five polynomials have



        • a negative value in $-1$, it is $-2018$ for the first four, and $-1$ for the last one,

        • a positive value in $0$, it is $+1$ for the first four, and bigger for the last one.

        So each polynomial has a root in the interval $[-1,0]$. The first derivative for each given polynomial is $>0$ on $Bbb R$, so the given polynomial functions are strictly increasing, from $-infty$ (when the argument tends to this limit) to $+infty$ (when the argument tends to this limit). So each given polynomial has the root in the interval $[-1,0]$.



      • The last polynomial has the root $-2018/2019<-9/10=-0.9$, and if we plug in this value, $0.9$, in any $P$ under the other four we get a negative number, since $0.9^13approx 0.25dots$, and $0.9^11approx 0.31dots$, so the term with the coefficient $2018$ "wins", $P(-0,9)<2018cdot frac 14+1<0$.


      Let now $ain(-1,0)$ be the root of
      $$ P=x^17 + 2018x^11 + 1 ,$$
      and let
      $$ Q=x^A + 2018x^B + 1$$
      be an other polynomial in the list, so $Ain17,19$,
      $Bin11,13$ and either $A>17$, or $B>11$.
      We have
      $$
      -1<a^11<a^13<a^17<a^19<0
      ,
      $$
      so
      $$Q(a) > P(a)=0 ,$$
      so the root of $Q$ is in the interval $[-1,a]$, being thus smaller than $a$.



      $square$




      Ths problem is solved, but it is always good to give a plot for the actors, in our case for the four polynomials, in the interval that hurts, here first $[-1,0]$.



      First plot, four polynomials in $[-1, 0]$



      ...then $[-0.6, 0]$.



      Second plot, four polynomials in $[-0.6, 0]$



      The following code was used:



      sage: p1, p2, p3, p4 = [ plot( p, (-1,0) ) for p in polynomials[:4] ]
      sage: (p1+p2+p3+p4).show()
      Launched png viewer for Graphics object consisting of 4 graphics primitives
      sage: p1, p2, p3, p4 = [ plot( p, (-0.6,0) ) for p in polynomials[:4] ]
      sage: (p1+p2+p3+p4).show()
      Launched png viewer for Graphics object consisting of 4 graphics primitives





      share|cite|improve this answer






















        up vote
        2
        down vote










        up vote
        2
        down vote









        Let us first use computer power to have the verdict, code:



        polynomials = ( x^19 + 2018*x^11 + 1, 
        x^17 + 2018*x^11 + 1,
        x^19 + 2018*x^13 + 1,
        x^17 + 2018*x^13 + 1,
        2019*x + 2018 )
        for p in polynomials:
        print ( "%s has only real root at %s"
        % ( p, p.roots(ring=AA, multiplicities=False)[0] ) )


        Results:



        x^19 + 2018*x^11 + 1 has only real root at -0.5006711240348649?
        x^17 + 2018*x^11 + 1 has only real root at -0.500670857826262?
        x^19 + 2018*x^13 + 1 has only real root at -0.556896901348879?
        x^17 + 2018*x^13 + 1 has only real root at -0.5568954928592386?
        2019*x + 2018 has only real root at -0.9995047052996532?


        (sage was used, free software for all mathematical purposes, it can even be "infected" with non-free software, if the user possesses the one or the other compatible choice.)



        Note: The above numbers are printed with a question mark at their end, but sage can does in fact exact computations with such numbers.



        And "the winner is..."



        x^17 + 2018*x^11 + 1


        Let us prove this by human means.




        • First of all, let us show that all roots are in the interval $[-1,0]$. Indeed, all five polynomials have



          • a negative value in $-1$, it is $-2018$ for the first four, and $-1$ for the last one,

          • a positive value in $0$, it is $+1$ for the first four, and bigger for the last one.

          So each polynomial has a root in the interval $[-1,0]$. The first derivative for each given polynomial is $>0$ on $Bbb R$, so the given polynomial functions are strictly increasing, from $-infty$ (when the argument tends to this limit) to $+infty$ (when the argument tends to this limit). So each given polynomial has the root in the interval $[-1,0]$.



        • The last polynomial has the root $-2018/2019<-9/10=-0.9$, and if we plug in this value, $0.9$, in any $P$ under the other four we get a negative number, since $0.9^13approx 0.25dots$, and $0.9^11approx 0.31dots$, so the term with the coefficient $2018$ "wins", $P(-0,9)<2018cdot frac 14+1<0$.


        Let now $ain(-1,0)$ be the root of
        $$ P=x^17 + 2018x^11 + 1 ,$$
        and let
        $$ Q=x^A + 2018x^B + 1$$
        be an other polynomial in the list, so $Ain17,19$,
        $Bin11,13$ and either $A>17$, or $B>11$.
        We have
        $$
        -1<a^11<a^13<a^17<a^19<0
        ,
        $$
        so
        $$Q(a) > P(a)=0 ,$$
        so the root of $Q$ is in the interval $[-1,a]$, being thus smaller than $a$.



        $square$




        Ths problem is solved, but it is always good to give a plot for the actors, in our case for the four polynomials, in the interval that hurts, here first $[-1,0]$.



        First plot, four polynomials in $[-1, 0]$



        ...then $[-0.6, 0]$.



        Second plot, four polynomials in $[-0.6, 0]$



        The following code was used:



        sage: p1, p2, p3, p4 = [ plot( p, (-1,0) ) for p in polynomials[:4] ]
        sage: (p1+p2+p3+p4).show()
        Launched png viewer for Graphics object consisting of 4 graphics primitives
        sage: p1, p2, p3, p4 = [ plot( p, (-0.6,0) ) for p in polynomials[:4] ]
        sage: (p1+p2+p3+p4).show()
        Launched png viewer for Graphics object consisting of 4 graphics primitives





        share|cite|improve this answer












        Let us first use computer power to have the verdict, code:



        polynomials = ( x^19 + 2018*x^11 + 1, 
        x^17 + 2018*x^11 + 1,
        x^19 + 2018*x^13 + 1,
        x^17 + 2018*x^13 + 1,
        2019*x + 2018 )
        for p in polynomials:
        print ( "%s has only real root at %s"
        % ( p, p.roots(ring=AA, multiplicities=False)[0] ) )


        Results:



        x^19 + 2018*x^11 + 1 has only real root at -0.5006711240348649?
        x^17 + 2018*x^11 + 1 has only real root at -0.500670857826262?
        x^19 + 2018*x^13 + 1 has only real root at -0.556896901348879?
        x^17 + 2018*x^13 + 1 has only real root at -0.5568954928592386?
        2019*x + 2018 has only real root at -0.9995047052996532?


        (sage was used, free software for all mathematical purposes, it can even be "infected" with non-free software, if the user possesses the one or the other compatible choice.)



        Note: The above numbers are printed with a question mark at their end, but sage can does in fact exact computations with such numbers.



        And "the winner is..."



        x^17 + 2018*x^11 + 1


        Let us prove this by human means.




        • First of all, let us show that all roots are in the interval $[-1,0]$. Indeed, all five polynomials have



          • a negative value in $-1$, it is $-2018$ for the first four, and $-1$ for the last one,

          • a positive value in $0$, it is $+1$ for the first four, and bigger for the last one.

          So each polynomial has a root in the interval $[-1,0]$. The first derivative for each given polynomial is $>0$ on $Bbb R$, so the given polynomial functions are strictly increasing, from $-infty$ (when the argument tends to this limit) to $+infty$ (when the argument tends to this limit). So each given polynomial has the root in the interval $[-1,0]$.



        • The last polynomial has the root $-2018/2019<-9/10=-0.9$, and if we plug in this value, $0.9$, in any $P$ under the other four we get a negative number, since $0.9^13approx 0.25dots$, and $0.9^11approx 0.31dots$, so the term with the coefficient $2018$ "wins", $P(-0,9)<2018cdot frac 14+1<0$.


        Let now $ain(-1,0)$ be the root of
        $$ P=x^17 + 2018x^11 + 1 ,$$
        and let
        $$ Q=x^A + 2018x^B + 1$$
        be an other polynomial in the list, so $Ain17,19$,
        $Bin11,13$ and either $A>17$, or $B>11$.
        We have
        $$
        -1<a^11<a^13<a^17<a^19<0
        ,
        $$
        so
        $$Q(a) > P(a)=0 ,$$
        so the root of $Q$ is in the interval $[-1,a]$, being thus smaller than $a$.



        $square$




        Ths problem is solved, but it is always good to give a plot for the actors, in our case for the four polynomials, in the interval that hurts, here first $[-1,0]$.



        First plot, four polynomials in $[-1, 0]$



        ...then $[-0.6, 0]$.



        Second plot, four polynomials in $[-0.6, 0]$



        The following code was used:



        sage: p1, p2, p3, p4 = [ plot( p, (-1,0) ) for p in polynomials[:4] ]
        sage: (p1+p2+p3+p4).show()
        Launched png viewer for Graphics object consisting of 4 graphics primitives
        sage: p1, p2, p3, p4 = [ plot( p, (-0.6,0) ) for p in polynomials[:4] ]
        sage: (p1+p2+p3+p4).show()
        Launched png viewer for Graphics object consisting of 4 graphics primitives






        share|cite|improve this answer












        share|cite|improve this answer



        share|cite|improve this answer










        answered Jun 12 at 17:24









        dan_fulea

        4,5601211




        4,5601211



























             

            draft saved


            draft discarded















































             


            draft saved


            draft discarded














            StackExchange.ready(
            function ()
            StackExchange.openid.initPostLogin('.new-post-login', 'https%3a%2f%2fmath.stackexchange.com%2fquestions%2f2814099%2f2018-amc-12a-problem-21-which-of-the-given-polynomials-has-the-greatest-real-ro%23new-answer', 'question_page');

            );

            Post as a guest













































































            這個網誌中的熱門文章

            How to combine Bézier curves to a surface?

            Mutual Information Always Non-negative

            Why am i infinitely getting the same tweet with the Twitter Search API?